Buscar

ÁLGEBRA CLÁSSICA

Faça como milhares de estudantes: teste grátis o Passei Direto

Esse e outros conteúdos desbloqueados

16 milhões de materiais de várias disciplinas

Impressão de materiais

Agora você pode testar o

Passei Direto grátis

Você também pode ser Premium ajudando estudantes

Faça como milhares de estudantes: teste grátis o Passei Direto

Esse e outros conteúdos desbloqueados

16 milhões de materiais de várias disciplinas

Impressão de materiais

Agora você pode testar o

Passei Direto grátis

Você também pode ser Premium ajudando estudantes

Faça como milhares de estudantes: teste grátis o Passei Direto

Esse e outros conteúdos desbloqueados

16 milhões de materiais de várias disciplinas

Impressão de materiais

Agora você pode testar o

Passei Direto grátis

Você também pode ser Premium ajudando estudantes
Você viu 3, do total de 78 páginas

Faça como milhares de estudantes: teste grátis o Passei Direto

Esse e outros conteúdos desbloqueados

16 milhões de materiais de várias disciplinas

Impressão de materiais

Agora você pode testar o

Passei Direto grátis

Você também pode ser Premium ajudando estudantes

Faça como milhares de estudantes: teste grátis o Passei Direto

Esse e outros conteúdos desbloqueados

16 milhões de materiais de várias disciplinas

Impressão de materiais

Agora você pode testar o

Passei Direto grátis

Você também pode ser Premium ajudando estudantes

Faça como milhares de estudantes: teste grátis o Passei Direto

Esse e outros conteúdos desbloqueados

16 milhões de materiais de várias disciplinas

Impressão de materiais

Agora você pode testar o

Passei Direto grátis

Você também pode ser Premium ajudando estudantes
Você viu 6, do total de 78 páginas

Faça como milhares de estudantes: teste grátis o Passei Direto

Esse e outros conteúdos desbloqueados

16 milhões de materiais de várias disciplinas

Impressão de materiais

Agora você pode testar o

Passei Direto grátis

Você também pode ser Premium ajudando estudantes

Faça como milhares de estudantes: teste grátis o Passei Direto

Esse e outros conteúdos desbloqueados

16 milhões de materiais de várias disciplinas

Impressão de materiais

Agora você pode testar o

Passei Direto grátis

Você também pode ser Premium ajudando estudantes

Faça como milhares de estudantes: teste grátis o Passei Direto

Esse e outros conteúdos desbloqueados

16 milhões de materiais de várias disciplinas

Impressão de materiais

Agora você pode testar o

Passei Direto grátis

Você também pode ser Premium ajudando estudantes
Você viu 9, do total de 78 páginas

Faça como milhares de estudantes: teste grátis o Passei Direto

Esse e outros conteúdos desbloqueados

16 milhões de materiais de várias disciplinas

Impressão de materiais

Agora você pode testar o

Passei Direto grátis

Você também pode ser Premium ajudando estudantes

Prévia do material em texto

1. ÁLGEBRA CLÁSSICA 
2. Avaliação Formativa I 
 
QUESTIONÁRIO 
Avaliação Formativa I 
Iniciado em Saturday, 17 Sep 2022, 10:02 
Estado Finalizada 
Concluída em Saturday, 17 Sep 2022, 10:12 
Tempo empregado 10 minutos 10 segundos 
Avaliar 3,00 de um máximo de 10,00(30%) 
Questão 1 
Correto 
Atingiu 1,00 de 1,00 
Marcar questão 
T
e
x
t
o 
d
a 
q
u
e
s
t
ã
o 
Determine a soma a + b + c sabendo 
que . 
 
 
 
Escolha uma opção: 
 
a. 
5 
https://virtual.uninta.edu.br/course/view.php?id=4379#section-5
 
b. 
7 
 
c. 
6 
 
d. 
4 
 
e. 
10 
F
e
e
d
b
a
c
k 
Sua resposta está correta. 
A resposta correta é: 4 
Questão 2 
Incorreto 
Atingiu 0,00 de 1,00 
Marcar questão 
T
e
x
t
o 
d
a 
q
u
e
s
t
ã
o 
Seja . Sabendo que é a inversa de A, calcule o valor de a + 
b + c + d é: 
 
 
 
Escolha uma opção: 
 
a. 
1 
 
b. 
3 
 
c. 
2 
 
d. 
4 
 
e. 
0 
F
e
e
d
b
a
c
k 
Sua resposta está incorreta. 
A resposta correta é: 0 
Questão 3 
Incorreto 
Atingiu 0,00 de 1,00 
Marcar questão 
T
e
x
t
o 
d
a 
q
u
e
s
t
ã
o 
Sobre sistema abaixo 
, podemos afirmar que é: 
 
 
 
Escolha uma opção: 
 
a. 
Possível e determinado 
 
b. 
Impossível 
 
c. 
Impossível e homogêneo 
 
d. 
Homogêneo 
 
e. 
Possível e indeterminado 
F
e
e
d
b
a
c
k 
Sua resposta está incorreta. 
A resposta correta é: Impossível 
Questão 4 
Incorreto 
Atingiu 0,00 de 1,00 
Marcar questão 
T
e
x
t
o 
d
a 
q
u
e
s
t
ã
o 
Seja o sistema: . Calcule k para que o sistema seja homogêneo. 
Escolha uma opção: 
 
a. 
-3 
 
b. 
-1 
 
c. 
-8 
 
d. 
9 
 
e. 
3 
F
e
e
d
b
a
c
k 
Sua resposta está incorreta. 
A resposta correta é: -3 
Questão 5 
Correto 
Atingiu 1,00 de 1,00 
Marcar questão 
T
e
x
t
o 
d
a 
q
u
e
s
t
ã
o 
Seja P a matriz inversa da matriz , então o elemento pertencente à 
segunda linha e à primeira coluna de P é igual a: 
 
 
 
Escolha uma opção: 
 
a. 
1/4 
 
b. 
4/3 
 
c. 
3/4 
 
d. 
-5/4 
 
e. 
7/4 
F
e
e
d
b
a
c
k 
Sua resposta está correta. 
A resposta correta é: -5/4 
Questão 6 
Incorreto 
Atingiu 0,00 de 1,00 
Marcar questão 
T
e
x
t
o 
d
a 
q
u
e
s
t
ã
o 
Dada a equação , ache α para que torne a sentença 
verdadeira. 
 
 
 
Escolha uma opção: 
 
a. 
1/2 
 
b. 
-1/5 
 
c. 
-8/5 
 
d. 
3/5 
 
e. 
3 
F
e
e
d
b
a
c
k 
Sua resposta está incorreta. 
A resposta correta é: -8/5 
Questão 7 
Correto 
Atingiu 1,00 de 1,00 
Marcar questão 
T
e
x
t
o 
d
a 
q
u
e
s
t
ã
o 
Determine m para que seja solução da equação . 
Escolha uma opção: 
 
a. 
-1 
 
b. 
0 
 
c. 
2 
 
d. 
1 
 
e. 
3 
F
e
e
d
b
a
c
k 
Sua resposta está correta. 
A resposta correta é: -1 
Questão 8 
Incorreto 
Atingiu 0,00 de 1,00 
Marcar questão 
T
e
x
t
o 
d
a 
q
u
e
s
t
ã
o 
Dada a matriz A = , calcule 3.a31 – 5.a42. 
 
 
 
Escolha uma opção: 
 
a. 
11 
 
b. 
20 
 
c. 
17 
 
d. 
25/2 
 
e. 
15 
F
e
e
d
b
a
c
k 
Sua resposta está incorreta. 
A resposta correta é: 11 
Questão 9 
Incorreto 
Atingiu 0,00 de 1,00 
Marcar questão 
T
e
x
t
o 
d
a 
q
u
e
s
t
ã
o 
Sejam as matrizes: 
 
 
. 
 
 
Se M = N, então o valor da soma de a + b + c é: 
 
 
 
Escolha uma opção: 
 
a. 
4 
 
b. 
5 
 
c. 
6 
 
d. 
3 
 
e. 
2 
F
e
e
d
b
a
c
k 
Sua resposta está incorreta. 
A resposta correta é: 5 
Questão 10 
Incorreto 
Atingiu 0,00 de 1,00 
Marcar questão 
T
e
x
t
o 
d
a 
q
u
e
s
t
ã
o 
Resolver o sistema ., temos: 
 
 
 
Escolha uma opção: 
 
a. 
 
 
b. 
 
 
c. 
 
 
d. 
 
 
e. 
 
F
e
e
d
b
a
c
k 
Sua resposta está incorreta. 
A resposta correta é: 
1. ÁLGEBRA CLÁSSICA 
2. Avaliação Formativa II 
 
QUESTIONÁRIO 
Avaliação Formativa II 
Iniciado em Saturday, 17 Sep 2022, 10:16 
https://virtual.uninta.edu.br/course/view.php?id=4379#section-5
Estado Finalizada 
Concluída em Saturday, 17 Sep 2022, 10:18 
Tempo empregado 1 minuto 38 segundos 
Avaliar 2,00 de um máximo de 10,00(20%) 
Questão 1 
Incorreto 
Atingiu 0,00 de 1,00 
Marcar questão 
T
e
x
t
o 
d
a 
q
u
e
s
t
ã
o 
Dada a matriz quadrada A, de ordem n, com , o valor 
de é: 
Escolha uma opção: 
 
a. 
3 
 
b. 
5 
 
c. 
2 
 
d. 
4 
 
e. 
1 
F
e
e
d
b
a
c
k 
Sua resposta está incorreta. 
A resposta correta é: 1 
Questão 2 
Incorreto 
Atingiu 0,00 de 1,00 
Marcar questão 
T
e
x
t
o 
d
a 
q
u
e
s
t
ã
o 
Dobrando-se o valor de cada elemento de uma matriz 3 x 3, o determinante: 
 
 
 
Escolha uma opção: 
 
a. 
Dobra 
 
b. 
Sextuplica 
 
c. 
Octuplica 
 
 
 
 
d. 
Não se altera 
 
e. 
Quadruplica 
 
 
 
F
e
e
d
b
a
c
k 
Sua resposta está incorreta. 
A resposta correta é: 
Octuplica 
Questão 3 
Incorreto 
Atingiu 0,00 de 1,00 
Marcar questão 
T
e
x
t
o 
d
a 
q
u
e
s
t
ã
o 
Considere as matrizes: 
 
 
 e 
 
 
O valor do determinante da matriz C = A⋅B é: 
 
 
 
Escolha uma opção: 
 
a. 
26 
 
b. 
16 
 
c. 
-6 
 
d. 
6 
 
e. 
10 
F
e
e
d
b
a
c
k 
Sua resposta está incorreta. 
A resposta correta é: 10 
Questão 4 
Incorreto 
Atingiu 0,00 de 1,00 
Marcar questão 
T
e
x
t
o 
d
a 
q
u
e
s
t
ã
o 
Sejam A e B matrizes 3 × 3 tais que det(A) = 3 e det(B) = 4. Então det(A × 2B) 
é igual a: 
 
 
 
Escolha uma opção: 
 
a. 
64 
 
b. 
48 
 
c. 
80 
 
d. 
96 
 
e. 
32 
F
e
e
d
b
a
c
k 
Sua resposta está incorreta. 
A resposta correta é: 96 
Questão 5 
Correto 
Atingiu 1,00 de 1,00 
Marcar questão 
T
e
x
t
o 
d
a 
q
u
e
s
t
ã
o 
Sabendo que , A= e B = , então det(A.B) vale: 
Escolha uma opção: 
 
a. 
-100 
 
b. 
100 
 
c. 
10 
 
d. 
20 
 
e. 
2 
F
e
e
d
b
a
c
k 
Sua resposta está correta. 
A resposta correta é: 100 
Questão 6 
Incorreto 
Atingiu 0,00 de 1,00 
Marcar questão 
T
e
x
t
o 
d
a 
q
u
e
s
t
ã
o 
Dada a matriz , cujo determinante vale – 3. Calcule o valor 
de 
 
 
 
Escolha uma opção: 
 
a. 
-48 
 
b. 
48 
 
c. 
-12 
 
d. 
6 
 
e. 
-6 
F
e
e
d
b
a
c
k 
Sua resposta está incorreta. 
A resposta correta é: 6 
Questão 7 
Incorreto 
Atingiu 0,00 de 1,00 
Marcar questão 
T
e
x
t
o 
d
a 
q
u
e
s
t
ã
o 
Sejam M e P matrizes quadradas de ordem 2. Se o valor do determinante da 
matriz M é 5 e P–1 = 3M, então o valor do determinante da matriz P é: 
 
 
 
Escolha uma opção: 
 
a. 
1/45 
 
b. 
45 
 
c. 
48 
 
d. 
1/15 
 
e. 
15 
F
e
e
d
b
a
c
k 
Sua resposta está incorreta. 
A resposta correta é: 1/15 
Questão 8 
Correto 
Atingiu 1,00 de 1,00 
Marcar questão 
T
e
x
t
o 
d
a 
q
u
e
s
t
ã
o 
Dada a matriz , calcule o valor de 
Escolha uma opção: 
 
a. 
-20 
 
b. 
-16 
 
c. 
80 
 
d. 
-80 
 
e. 
16 
F
e
e
d
b
a
c
k 
Sua resposta está correta. 
A resposta correta é: -80 
Questão 9 
Incorreto 
Atingiu 0,00 de 1,00 
Marcar questão 
T
e
x
t
o 
d
a 
q
u
e
s
t
ã
o 
Resolva a equação = - 6. 
 
 
 
Escolha uma opção: 
 
a. 
6 e 1 
 
b. 
3 e 5 
 
c. 
-2 e -3 
 
d. 
-2 e 3 
 
e. 
2 e 3 
F
e
e
d
b
a
c
k 
Sua resposta está incorreta. 
A resposta correta é: 2 e 3 
Questão 10 
Incorreto 
Atingiu 0,00 de 1,00 
Marcar questão 
T
e
x
t
o 
d
a 
q
u
e
s
t
ã
o 
Resolva a equação . 
 
 
 
Escolha uma opção: 
 
a. 
-22 
 
b. 
22 
 
c. 
4 
 
d. 
11 
 
e. 
-11 
F
e
e
d
b
a
c
k 
Sua resposta está incorreta. 
A resposta correta é: -22 
1. ) 
 
QUESTIONÁRIO 
AP3.1 - Avaliação Formativa 
Processual - Online (vale 40% da 
MAP) 
Iniciado em Saturday, 17 Sep 2022, 10:18 
Estado Finalizada 
Concluída em Saturday, 17 Sep 2022, 10:29 
Tempo empregado 10 minutos 40 segundos 
Notas 9,00/20,00 
Avaliar4,50 de um máximo de 10,00(45%) 
Questão 1 
Incorreto 
Atingiu 0,00 de 1,00 
Marcar questão 
T
e
x
t
o 
d
a 
q
u
e
s
t
ã
o 
Se a tríade (x, y, z) é solução do sistema linear abaixo: 
 
 
. Calcule x + y + z. 
 
 
 
Escolha uma opção: 
 
a. 
3 
 
b. 
1 
 
c. 
4 
 
d. 
5 
 
e. 
2 
F
e
e
d
b
a
c
k 
Sua resposta está incorreta. 
A resposta correta é: 3 
Questão 2 
Incorreto 
Atingiu 0,00 de 1,00 
Marcar questão 
T
e
x
t
o 
d
a 
q
u
e
s
t
ã
o 
Calcule o valor do determinante da matriz A = . 
 
 
 
Escolha uma opção: 
 
a. 
Sen(2x) 
 
b. 
0 
 
c. 
3 
 
d. 
-1 
 
e. 
1 
F
e
e
d
b
a
c
k 
Sua resposta está incorreta. 
A resposta correta é: 1 
Questão 3 
Correto 
Atingiu 1,00 de 1,00 
Remover marcação 
T
e
x
t
o 
d
a 
q
u
e
s
t
ã
o 
Sejam A e B matrizes 3 × 3 tais que det(A) = 3 e det(B) = 4. Então det(A × 2B) 
é igual a: 
 
 
 
Escolha uma opção: 
 
a. 
64 
 
b. 
80 
 
c. 
32 
 
d. 
48 
 
e. 
96 
F
e
e
d
b
a
c
k 
Sua resposta está correta. 
A resposta correta é: 96 
Questão 4 
Correto 
Atingiu 1,00 de 1,00 
Marcar questão 
T
e
x
t
o 
d
a 
q
u
e
s
t
ã
o 
Determine a soma dos elementos das matrizes A e B, ambas (2x2) cujos 
elementos foram dados por: 
 
 
 
 
 
 
 
Escolha uma opção: 
 
a. 
7 
 
b. 
10 
 
c. 
5 
 
d. 
20 
 
e. 
15 
F
e
e
d
b
a
c
k 
Sua resposta está correta. 
A resposta correta é: 7 
Questão 5 
Correto 
Atingiu 1,00 de 1,00 
Remover marcação 
T
e
x
t
o 
d
a 
q
u
e
s
t
ã
o 
Uma matriz A quadrada, de ordem 4, possui determinante igual a 2. O valor 
de é: 
 
 
 
Escolha uma opção: 
 
a. 
16 
 
b. 
4 
 
c. 
32 
 
d. 
64 
 
e. 
8 
F
e
e
d
b
a
c
k 
Sua resposta está correta. 
A resposta correta é: 32 
Questão 6 
Correto 
Atingiu 1,00 de 1,00 
Remover marcação 
T
e
x
t
o 
d
a 
q
u
e
s
t
ã
o 
Resolvendo o sistema , temos a seguinte solução: 
Escolha uma opção: 
 
a. 
 
 
b. 
 
 
c. 
 
 
d. 
 
 
e. 
 
F
e
e
d
b
a
c
k 
Sua resposta está correta. 
A resposta correta é: 
Questão 7 
Incorreto 
Atingiu 0,00 de 1,00 
Remover marcação 
T
e
x
t
o 
d
a 
q
u
e
s
t
ã
o 
Calcule o determinante da matriz P2, em que P é a matriz P = . 
 
 
 
Escolha uma opção: 
 
a. 
2 
 
b. 
0 
 
c. 
64 
 
d. 
4 
 
e. 
8 
F
e
e
d
b
a
c
k 
Sua resposta está incorreta. 
A resposta correta é: 64 
Questão 8 
Incorreto 
Atingiu 0,00 de 1,00 
Marcar questão 
T
e
x
t
o 
d
a 
q
u
e
s
t
ã
o 
Determine o valor de a para que o sistema seja possível e 
determinado (SPD). 
 
 
 
Escolha uma opção: 
 
a. 
a ≠ -8/5 
 
b. 
a = 3/5 
 
c. 
a ≠ 3/2 
 
d. 
a ≠ – 1/2 
 
e. 
a = – 1/5 
F
e
e
d
b
a
c
k 
Sua resposta está incorreta. 
A resposta correta é: a ≠ – 1/2 
Questão 9 
Correto 
Atingiu 1,00 de 1,00 
Remover marcação 
T
e
x
t
o 
d
a 
q
u
e
s
t
ã
o 
Calcule o determinante da matriz : 
 
 
 
Escolha uma opção: 
 
a. 
200 
 
b. 
-97 
 
c. 
97 
 
d. 
-200 
 
e. 
297 
F
e
e
d
b
a
c
k 
Sua resposta está correta. 
A resposta correta é: 97 
Questão 10 
Correto 
Atingiu 1,00 de 1,00 
Remover marcação 
T
e
x
t
o 
d
a 
q
u
e
s
t
ã
o 
13. O valor de a para que a igualdade matricial seja 
verdadeira é: 
 
 
 
Escolha uma opção: 
 
a. 
-2 
 
b. 
1 
 
c. 
2 
 
d. 
-1 
 
e. 
0 
F
e
e
d
b
a
c
k 
Sua resposta está correta. 
A resposta correta é: 2 
Questão 11 
Correto 
Atingiu 1,00 de 1,00 
Remover marcação 
T
e
x
t
o 
d
a 
q
u
e
s
t
ã
o 
Seja P a matriz inversa da matriz , então o elemento pertencente à 
segunda linha e à primeira coluna de P é igual a: 
 
 
 
Escolha uma opção: 
 
a. 
4/3 
 
b. 
-5/4 
 
c. 
1/4 
 
d. 
3/4 
 
e. 
7/4 
F
e
e
d
b
a
c
k 
Sua resposta está correta. 
A resposta correta é: -5/4 
Questão 12 
Incorreto 
Atingiu 0,00 de 1,00 
Marcar questão 
T
e
x
t
o 
d
a 
q
u
e
s
t
ã
o 
Sejam A, B e C matrizes quadradas quaisquer de ordem três e os números reais 
α e β não nulos. É falsa a afirmativa: 
 
 
 
Escolha uma opção: 
 
a. 
(A+B).C = C . (A+B) 
 
 
 
 
b. 
1.A = A.1 = A 
 
c. 
α . A+ β.A = (α + β ).A 
 
 
 
 
d. 
(A+B) +C = A+(B+C) 
 
 
 
 
e. 
(A.B).C = A .(B.C) 
F
e
e
d
b
a
c
k 
Sua resposta está incorreta. 
A resposta correta é: 
(A+B).C = C . (A+B) 
Questão 13 
Incorreto 
Atingiu 0,00 de 1,00 
Marcar questão 
T
e
x
t
o 
d
a 
q
u
e
s
t
ã
o 
Sabendo que det A = 3. Sabendo que A tem ordem 4, calcule det (3A): 
 
 
 
Escolha uma opção: 
 
a. 
-81 
 
b. 
27 
 
c. 
243 
 
d. 
81 
 
e. 
-243 
F
e
e
d
b
a
c
k 
Sua resposta está incorreta. 
A resposta correta é: 243 
Questão 14 
Incorreto 
Atingiu 0,00 de 1,00 
Marcar questão 
T
e
x
t
o 
d
a 
q
u
e
s
t
ã
o 
Sabendo que as matrizes são iguais, determine o valor de X + Y: 
 
 
 
 
Escolha uma opção: 
 
a. 
5 
 
b. 
8 
 
c. 
10 
 
d. 
7 
 
e. 
6 
F
e
e
d
b
a
c
k 
Sua resposta está incorreta. 
A resposta correta é: 6 
Questão 15 
Incorreto 
Atingiu 0,00 de 1,00 
Marcar questão 
T
e
x
t
o 
d
a 
q
u
e
s
t
ã
o 
Seja A = (aij)3x3 tal que aij = i – j. Calcule det A: 
Escolha uma opção: 
 
a. 
1 
 
b. 
4 
 
c. 
8 
 
d. 
-1 
 
e. 
0 
F
e
e
d
b
a
c
k 
Sua resposta está incorreta. 
A resposta correta é: 0 
Questão 16 
Incorreto 
Atingiu 0,00 de 1,00 
Marcar questão 
T
e
x
t
o 
d
a 
q
u
e
s
t
ã
o 
Dada as matrizes e . O valor de x2 + y2 + w2 – z é: 
 
 
 
Escolha uma opção: 
 
a. 
45 
 
b. 
41 
 
c. 
31 
 
d. 
39 
 
e. 
29 
F
e
e
d
b
a
c
k 
Sua resposta está incorreta. 
A resposta correta é: 39 
Questão 17 
Incorreto 
Atingiu 0,00 de 1,00 
Marcar questão 
T
e
x
t
o 
d
a 
q
u
e
s
t
ã
o 
Se , então vale: 
Escolha uma opção: 
 
a. 
4 
 
b. 
6 
 
c. 
3 
 
d. 
7 
 
e. 
5 
F
e
e
d
b
a
c
k 
Sua resposta está incorreta. 
A resposta correta é: 5 
Questão 18 
Incorreto 
Atingiu 0,00 de 1,00 
Marcar questão 
T
e
x
t
o 
d
a 
q
u
e
s
t
ã
o 
O determinante da matriz , onde P, Q, R e S são os determinantes das 
matrizes 2 × 2. 
 , , e 
É igual a: 
 
 
 
Escolha uma opção: 
 
a. 
0 
 
b. 
2 
 
c. 
1 
 
d. 
-1 
 
e. 
3 
F
e
e
d
b
a
c
k 
Sua resposta está incorreta. 
A resposta correta é: 0 
Questão 19 
Correto 
Atingiu 1,00 de 1,00 
Remover marcação 
T
e
x
t
o 
d
a 
q
u
e
s
t
ã
o 
Seja o sistema: . Calcule k para que o sistema seja homogêneo. 
Escolha uma opção: 
 
a. 
-1 
 
b. 
-8 
 
c. 
-3 
 
d. 
9 
 
e. 
3 
F
e
e
d
b
a
c
k 
Sua resposta está correta. 
A resposta correta é: -3 
Questão 20 
Correto 
Atingiu 1,00 de 1,00 
Remover marcação 
T
e
x
t
o 
d
a 
q
u
e
s
t
ã
o 
Determine o valor de x real para que o determinante da matriz seja 
nulo: 
 
 
 
Escolha uma opção: 
 
a. 
1 
 
b. 
0 ou 
 
c. 
2 
 
d. 
0 ou -1 
 
e. 
0 ou 1 
F
e
e
d
b
a
c
k 
Sua resposta está correta. 
A resposta correta é: 0 ou 
 
Iniciado em Saturday, 17 Sep 2022, 10:39 
Estado Finalizada 
Concluída em Saturday, 17 Sep 2022, 10:47 
Tempo empregado 8 minutos 40 segundos 
Notas 15,00/20,00 
Avaliar 7,50 de um máximo de 10,00(75%) 
Questão 1 
Incorreto 
Atingiu 0,00 de 1,00 
Marcar questão 
T
e
x
t
o 
d
a 
q
u
e
s
t
ã
o 
Considere as matrizes e . Calcule (2Dt – 3Et) 
t. 
 
 
 
Escolha uma opção: 
 
a. 
 
 
b. 
 
 
c. 
 
 
d. 
 
 
e. 
 
F
e
e
d
b
a
c
k 
Sua resposta está incorreta. 
A resposta correta é: 
Questão 2 
Correto 
Atingiu 1,00 de 1,00 
Remover marcação 
T
e
x
t
o 
d
a 
q
u
e
s
t
ã
o 
Determine o valor de x real para que o determinanteda matriz seja 
nulo: 
 
 
 
Escolha uma opção: 
 
a. 
0 ou 1 
 
b. 
0 ou -1 
 
c. 
1 
 
d. 
0 ou 
 
e. 
2 
F
e
e
d
b
a
c
k 
Sua resposta está correta. 
A resposta correta é: 0 ou 
Questão 3 
Correto 
Atingiu 1,00 de 1,00 
Remover marcação 
T
e
x
t
o 
d
a 
q
u
e
s
t
ã
o 
Sabendo que det A = 3. Sabendo que A tem ordem 4, calcule det (3A): 
 
 
 
Escolha uma opção: 
 
a. 
-243 
 
b. 
243 
 
c. 
81 
 
d. 
-81 
 
e. 
27 
F
e
e
d
b
a
c
k 
Sua resposta está correta. 
A resposta correta é: 243 
Questão 4 
Correto 
Atingiu 1,00 de 1,00 
Remover marcação 
T
e
x
t
o 
d
a 
q
u
e
s
t
ã
o 
Calcule o valor do determinante da matriz A = . 
 
 
 
Escolha uma opção: 
 
a. 
0 
 
b. 
1 
 
c. 
-1 
 
d. 
3 
 
e. 
Sen(2x) 
F
e
e
d
b
a
c
k 
Sua resposta está correta. 
A resposta correta é: 1 
Questão 5 
Correto 
Atingiu 1,00 de 1,00 
Marcar questão 
T
e
x
t
o 
d
a 
q
u
e
s
t
ã
o 
Um estádio recebeu um público pagante de jogo 3150 pessoas para uma 
partida de futebol. O preço da arquibancada (geral) era R$ 10,00 e o da 
cadeira numerada, R$ 30,00. Se a renda foi de R$ 53.900,00, a quantidade de 
pessoas que usaram cadeiras numeradas foi: 
 
 
 
Escolha uma opção: 
 
a. 
2930 
 
b. 
2820 
 
c. 
2120 
 
d. 
1120 
 
e. 
2030 
F
e
e
d
b
a
c
k 
Sua resposta está correta. 
A resposta correta é: 1120 
Questão 6 
Correto 
Atingiu 1,00 de 1,00 
Marcar questão 
T
e
x
t
o 
d
a 
q
u
e
s
t
ã
o 
Seja a um número real. Considere os sistemas lineares em x, y e z. Calcule o 
valor de a para que o sistema admita infinitas soluções: 
 
 
 
Escolha uma opção: 
 
a. 
a ≠ – 1/2 
 
 
 
 
b. 
a = 3/5 
 
 
 
 
c. 
a = 1/2 
 
 
 
 
d. 
a ≠ 3/2 
 
e. 
a = – 1/5 
F
e
e
d
b
a
c
k 
Sua resposta está correta. 
A resposta correta é: 
a = 1/2 
Questão 7 
Incorreto 
Atingiu 0,00 de 1,00 
Marcar questão 
T
e
x
t
o 
d
a 
q
u
e
s
t
ã
o 
Resolva a equação = 3 
Escolha uma opção: 
 
a. 
-3 
 
b. 
14 
 
c. 
5 
 
d. 
1 
 
e. 
-5 
F
e
e
d
b
a
c
k 
Sua resposta está incorreta. 
A resposta correta é: -5 
Questão 8 
Correto 
Atingiu 1,00 de 1,00 
Remover marcação 
T
e
x
t
o 
d
a 
q
u
e
s
t
ã
o 
Dada as matrizes e . O valor de x2 + y2 + w2 – z é: 
 
 
 
Escolha uma opção: 
 
a. 
41 
 
b. 
31 
 
c. 
39 
 
d. 
45 
 
e. 
29 
F
e
e
d
b
a
c
k 
Sua resposta está correta. 
A resposta correta é: 39 
Questão 9 
Correto 
Atingiu 1,00 de 1,00 
Remover marcação 
T
e
x
t
o 
d
a 
q
u
e
s
t
ã
o 
Dada a matriz A = , calcule 3.a31 – 5.a42. 
 
 
 
Escolha uma opção: 
 
a. 
25/2 
 
b. 
15 
 
c. 
17 
 
d. 
11 
 
e. 
20 
F
e
e
d
b
a
c
k 
Sua resposta está correta. 
A resposta correta é: 11 
Questão 10 
Correto 
Atingiu 1,00 de 1,00 
Remover marcação 
T
e
x
t
o 
d
a 
q
u
e
s
t
ã
o 
Dada a matriz , calcule o valor de 
Escolha uma opção: 
 
a. 
80 
 
b. 
16 
 
c. 
-16 
 
d. 
-80 
 
e. 
-20 
F
e
e
d
b
a
c
k 
Sua resposta está correta. 
A resposta correta é: -80 
Questão 11 
Correto 
Atingiu 1,00 de 1,00 
Marcar questão 
T
e
x
t
o 
d
a 
q
u
e
s
t
ã
o 
Resolver a equação = 0 
Escolha uma opção: 
 
a. 
2 e 3 
 
b. 
0 e 2 
 
c. 
0 e 4 
 
d. 
1 e 4 
 
e. 
0 e 8 
F
e
e
d
b
a
c
k 
Sua resposta está correta. 
A resposta correta é: 0 e 4 
Questão 12 
Correto 
Atingiu 1,00 de 1,00 
Remover marcação 
T
e
x
t
o 
d
a 
q
u
e
s
t
ã
o 
Sobre sistema abaixo 
, podemos afirmar que é: 
 
 
 
Escolha uma opção: 
 
a. 
Impossível e homogêneo 
 
b. 
Possível e indeterminado 
 
c. 
Homogêneo 
 
d. 
Impossível 
 
e. 
Possível e determinado 
F
e
e
d
b
a
c
k 
Sua resposta está correta. 
A resposta correta é: Impossível 
Questão 13 
Correto 
Atingiu 1,00 de 1,00 
Remover marcação 
T
e
x
t
o 
d
a 
q
u
e
s
t
ã
o 
Considere as matrizes: 
 
 
 e 
 
 
O valor do determinante da matriz C = A⋅B é: 
 
 
 
Escolha uma opção: 
 
a. 
16 
 
b. 
10 
 
c. 
-6 
 
d. 
26 
 
e. 
6 
F
e
e
d
b
a
c
k 
Sua resposta está correta. 
A resposta correta é: 10 
Questão 14 
Correto 
Atingiu 1,00 de 1,00 
Remover marcação 
T
e
x
t
o 
d
a 
q
u
e
s
t
ã
o 
Seja . Sabendo que é a inversa de A, calcule o valor de a + 
b + c + d é: 
 
 
 
Escolha uma opção: 
 
a. 
0 
 
b. 
1 
 
c. 
3 
 
d. 
2 
 
e. 
4 
F
e
e
d
b
a
c
k 
Sua resposta está correta. 
A resposta correta é: 0 
Questão 15 
Incorreto 
Atingiu 0,00 de 1,00 
Marcar questão 
T
e
x
t
o 
d
a 
q
u
e
s
t
ã
o 
A soma dos valores de x e de y que satisfazem as equações – 2 = 0 e 3x – 
2y = 0 é igual a: 
 
 
 
Escolha uma opção: 
 
a. 
8 
 
b. 
5 
 
c. 
6 
 
d. 
4 
 
e. 
3 
F
e
e
d
b
a
c
k 
Sua resposta está incorreta. 
A resposta correta é: 5 
Questão 16 
Incorreto 
Atingiu 0,00 de 1,00 
Marcar questão 
T
e
x
t
o 
d
a 
q
u
e
s
t
ã
o 
Dada a matriz A= , calcule o determinante da matriz inversa de A. 
Escolha uma opção: 
 
a. 
0 
 
b. 
1 
 
c. 
-1 
 
d. 
2 
 
e. 
-2 
F
e
e
d
b
a
c
k 
Sua resposta está incorreta. 
A resposta correta é: -1 
Questão 17 
Correto 
Atingiu 1,00 de 1,00 
Remover marcação 
T
e
x
t
o 
d
a 
q
u
e
s
t
ã
o 
Determine m para que seja solução da equação . 
Escolha uma opção: 
 
a. 
3 
 
b. 
2 
 
c. 
1 
 
d. 
-1 
 
e. 
0 
F
e
e
d
b
a
c
k 
Sua resposta está correta. 
A resposta correta é: -1 
Questão 18 
Incorreto 
Atingiu 0,00 de 1,00 
Marcar questão 
T
e
x
t
o 
d
a 
q
u
e
s
t
ã
o 
Determine os valores de a e b de modo 
que . 
 
 
 
Escolha uma opção: 
 
a. 
 
 
b. 
 
 
c. 
 
 
d. 
 
 
e. 
 
F
e
e
d
b
a
c
k 
Sua resposta está incorreta. 
A resposta correta é: 
Questão 19 
Correto 
Atingiu 1,00 de 1,00 
Marcar questão 
T
e
x
t
o 
d
a 
q
u
e
s
t
ã
o 
Dadas as matrizes , e calcule a 
matriz X na equação matricial X – M = N – P: 
 
 
 
Escolha uma opção: 
 
a. 
 
 
b. 
 
 
c. 
 
 
d. 
 
 
e. 
 
F
e
e
d
b
a
c
k 
Sua resposta está correta. 
A resposta correta é: 
Questão 20 
Correto 
Atingiu 1,00 de 1,00 
Remover marcação 
T
e
x
t
o 
d
a 
q
u
e
s
t
ã
o 
13. O valor de a para que a igualdade matricial seja 
verdadeira é: 
 
 
 
Escolha uma opção: 
 
a. 
-1 
 
b. 
-2 
 
c. 
1 
 
d. 
2 
 
e. 
0 
F
e
e
d
b
a
c
k 
Sua resposta está correta. 
A resposta correta é: 2 
Terminar revisão 
 
https://virtual.uninta.edu.br/mod/quiz/view.php?id=555262

Continue navegando